You are on page 1of 162

PPE Headquarters

FE
CIVIL

Practice Problems
• 110 exam questions
• Complete with answers and explanations
Chapter 1

Mathematics

A.Geometry
B. Calculus
C. Equation Roots
D. Vectors
!
1. A$circle$is$formed$by$applying$this$following$equation:$
x 2 − 16 x + y 2 + 8 y + 71 = 0 $
Find$the$coordinates$of$its$center.$
(A)$(>8,4)$$
(B)$(8,>4)$$
(C)$(4,>8)$
(D)$(>4,8)$
$
$
$
$
$
2. Determine$the$roots$of$ x3 − 2 x 2 − 5 x + 6 = 0 .$
(A)$1,$2,$3$
(B)$>1,$2,$>3$
(C)$1,$>2,$3$
(D)$>1,$>2,$>3$
$
$
$
$
$

2
3. Determine$ x, y, z $satisfying$these$three$simultaneous$equations.$
$
x + y − z = −1
x− y−z =3 $
x+ y+z =5
(A)$ ( −4, 2, −3) $$

(B)$$ ( 4, 2, −3) $$

(C)$ ( 4,-2,3) $$

(D)$ ( −4, −2,3) $$

$
$
$
4. !Determine$the$length$of$a$straight$line$which$connects$point$ A (3, −1, −4) $and$point$ B ( −1, −5, 2 ) $at$both$ends$of$the$line.$

(A)$ 17 $

(B)$ 2 17 $

(C)$ 3 17 $

(D)$ 4 17 $$
$
$
$
$

3
5. Determine$the$angle$ θ $between$these$two$vectors.$
$
V1 = 4i + j + 6k
$
V2 = i + 3 j + 4k

(A)$28.53°$
(B)$33.37°$
(C)$38.53°$
(D)$43.37°$
$
$
$
$
6. Referring$to$the$vectors$from$question$No.$5,$find$the$cross$product$of$those$two$vectors.$
(A)$ −14i + 10 j + 11k $$
(B)$ -14i -10 j + 11k $
(C)$ 14i + 10 j − 11k $
(D)$ 14i − 10 j − 11k $
$
$
$
$
$

4
6 x 2 − 3x + 20
7. Solve$this$equation$ lim $$
x →∞ 3x 2 + 5 x
(A)$1$
(B)$2$
(C)$3$
(D)$4$
$
$
$
$
8. Find$the$inflection$point$for$the$polynomial$function$as$follows$
f ( x ) = x3 − 6 x2 + 12 x − 5$

(A)$ ( −2, −3) $

(B)$ ( 2, −3)$

(C)$ ( −2,3) $

(D)$ ( 2,3) $

$
$
$
$
$

5
"2 ≤ x ≤ 4
9. Solve$this$integral$ ∫∫ xydxdy $where$ R = # $$
R $4 ≤ y ≤ 6
(A)$8$
(B)$10$
(C)$12$
(D)$16$
$
$
$
$
$
10. A$straight$line$intercepts$Y$axis$at$ y = 4 $and$also$passes$point$ P ( 2, 2 ) .$Determine$the$slope$of$the$line.!

(A)$>2$
(B)$>1$
(C)$1$
(D)$2$
$
! !

6
$
$ $

7
1.$The$correct$answer$is$(B).$
$
The$general$equation$of$a$circle$is:$
2 2
( x − a ) + ( y − b) = r2 $

Where$ ( a, b ) $is$the$coordinates$of$its$center$and$ r $is$the$radius$of$the$circle.$

Therefore,$
x 2 − 16 x + y 2 + 8 y + 71 = 0 $
$
x2 − 16 x + 64 + y 2 + 8 y + 16 = 9 $
$
2 2
( x − 8) + ( y + 4 ) = 32 $
$
By$following$the$general$equation$of$a$circle,$the$coordinates$of$its$center$can$be$determined$as$ (8, −4 ) .$

$
$
2.$The$correct$answer$is$(C).$
x3 − 2 x 2 − 5 x + 6 = 0 $

(x 2
)
+ x − 2 ( x − 3) = 0 $$

( x −1)( x + 2)( x − 3) = 0 $
The$roots$are$1,$>2,$3$
$

8
3.$The$correct$answer$is$(C).$
Add$the$first$and$second$equations$to$find$ y .$The$ x $and$ z $terms$cancel.$
$
2 y = −4 → y = −2$$
$
Subtract$the$first$and$third$equations$to$find$ z .$The$ x $and$ y $terms$cancel.$
$
−2z = −6 → z = 3 $
$
Substitute$ y = −2 $and$ z = 3 $to$one$of$the$equations$to$find$ x .$
x + y + z = 5$$
$
x + ( −2 ) + 3 = 5
$
$
x = 4$
$
4.$The$correct$answer$is$(B).$
$
2 2 2
AB = ( −1 − 3) + ( −5 + 1) + ( 2 + 4 ) = 68 = 2 17 $$

$
$
$

9
5.$The$correct$answer$is$(B).$
# V1 ⋅V2 $
θ = cos −1 && ''
( V1 V2 )
# $
−1 &
θ = cos &
( 4 )(1) + (1)(3) + ( 6 )( 4 ) '
'$
( 2
& 4 +1 + 6
(
2 2
)(
12 + 32 + 42 ) '
)
# 31 $
θ = cos −1 & '
( 53 26 )
θ = 33.37°
$
$
$
$
6.$The$correct$answer$is$(B).$
i j k
V1 × V2 = 4 1 6
1 3 4
V1 × V2 = ( (1)( 4 ) − ( 6 )( 3) ) i − ( ( 4 )( 4 ) − ( 6 )(1) ) j + ( ( 4 )(3) − (1)(1) ) k $
V1 × V2 = −14i − 10 j + 11k
$
$
$
$
$

10
7.$The$correct$answer$is$(B).$

Note$that$the$denominator$ 3 x 2 + 5 x $leads$to$an$indeterminate$form$ $.$To$avoid$this,$divide$the$function$as$a$whole$by$ x 2 $so$that:$

$
1 3 20 3 20
6− + 2 6− +
6 x 2 − 3 x + 20 x 2 x x = lim ∞ ∞ = 6 = 2$
lim = lim
x →∞ 2
3x + 5 x 1 x→∞ 5 x →∞ 5 3
2
3+ 3+
x x ∞
$
$
$
8.$The$correct$answer$is$(D).$
Inflection$point$occurs$when$second$derivation$of$ f ( x ) $is$0.$

$
f ( x ) = x3 − 6 x 2 + 12 x − 5
f ' ( x ) = 3x 2 − 12 x + 12
$
f " ( x ) = 6 x − 12 = 0 → x = 2
2
f ( 2 ) = 23 − 6 ( 2 ) + 12 ( 2 ) − 5 = 3
$
Therefore,$the$inflection$point$is$ ( 2,3) $$

$
$
$
$
11
9.$The$correct$answer$is$(D).$
If$the$region$ R $can$be$split$into$two$separate$regions$ R1 $and$ R2 ,$then$the$integral$can$be$written$as:$

∫∫ xydxdy = ∫∫ xydxdy + ∫∫ xydxdy


R R1 R2
4 6
1 2 1
∫∫ xydxdy = x + y2
R
2 2 2 4 $
∫∫ xydxdy = 6 + 10
R

∫∫ xydxdy = 16
R

$
$
$
10.$The$correct$answer$is$(B).$
The$general$equation$of$straight$line$is$ y = mx + b $where$ m $is$the$slope$of$the$line.$
y = mx + b
2 = m (2) + 4
$
2 m = −2
m = −1
$
! !

12
Chapter 2

Probability
and Statistics

A. Central Tendencies & Dispersions


B. Single Mean Estimations
C. Curve Fitting & Regression
D. Weighted Average Decisions
11.$The$bodyweight$of$students$in$the$elementary$school$are$summarized$in$the$following$table.$Determine$the$mean$of$the$
bodyweight.$
Bodyweight$ Frequency$
(kg)$
27$ 3$
30$ 5$
32$ 8$
33$ 14$
36$ 10$
40$ 6$
45$ 7$
(A)$33.82$
(B)$35.17$
(C)$35.64$
(D)$36.13$
$
$
12.$Referring$to$the$previous$question,$find$the$mode$of$the$bodyweight.$
(A)$32$
(B)$33$
(C)$36$
(D)$40$
$

14
13.$ A$result$of$marathon$competition$is$recorded$to$determine$the$statistic$result.$This$data$represents$the$time$spent$to$finish$the$
marathon:$54$minutes,$50$minutes,$56$minutes,$61$minutes,$and$51$minutes.$Find$the$variance$of$the$recorded$data.$
(A)$17.3$
(B)$18.3$
(C)$19.3$
(D)$20.3$
$
$
$
$
14.$Find$the$standard$deviation$of${5,$10,$3,$8,$12}.$
(A)$10.87$
(B)$11.65$
(C)$13.26$
(D)$14.54$
$
$
$
$
$
$
$
$

15
15.$ A$dice$is$rolled$and$it$shows$number$3.$Find$the$probability$the$dice$shows$number$3$again$if$it$is$rolled$once$more.$
1
(A)$ $$
2
1
(B)$ $
6
1
(C)$ $
12
1
(D)$ $
3
$
$
$
$
$
16.$ An$observation$is$made$to$check$the$quality$of$books$produced$at$the$publisher.$It$is$found$that$5%$of$the$total$books$are$
defected.$Determine$the$probability$the$publisher$picks$up$3$defected$books$of$total$10$books$chosen$randomly.$
(A)$0.0105$
(B)$0.0205$
(C)$0.0305$
(D)$0.0405$
$ $
$
$
$ $

16
$ $

17
11.$The$correct$answer$is$(B).$
The$mean$is$determined$by$dividing$the$sum$of$the$data$by$the$total$data.$
$

x=
( 27 )(3) + (30 )(5) + (32 )(8) + (33)(14 ) + (36 )(10 ) + ( 40 )(6 ) + ( 45)(7 ) = 35.17 $
3 + 5 + 8 + 14 + 10 + 6 + 7
$
$
$
$
12.$The$correct$answer$is$(B).$
The$mode$is$the$most$frequent$data$among$all$recorded$data.$We$can$determine$the$mode$directly$by$choosing$the$most$
frequent$data,$which$is$33$kg.$
$
$
13.$The$correct$answer$is$(C).$
From$5$recorded$data$in$the$question,$first$we$need$to$find$the$mean$then$we$can$get$the$variance.$

x=
∑x i
=
54 + 50 + 56 + 61 + 51
= 54.4
n 5
2

σ=
∑( x − x) i
$
n −1
2 2 2 2 2

σ=
(54 − 54.4 ) + (50 − 54.4 ) + (56 − 54.4 ) + (61 − 54.4 ) + (51 − 54.4 )
5 −1
σ = 19.3
$

18
14.$The$correct$answer$is$(C).$
We$can$calculate$the$standard$deviation$by$calculating$the$square$root$of$variance.$
$

x=
∑x i
=
5 + 10 + 3 + 8 + 12
= 19
n 5
2

σ=
∑ ( x − x) i

n −1 $
2 2 2 2 2

σ=
(5 − 19 ) + (10 − 19 ) + (3 − 19 ) + (8 − 19 ) + (12 − 19 )
5 −1
σ = 13.26
$
$
$
$
15.$The$correct$answer$is$(B).$
The$sample$space$of$the$rolling$dice$is$ {1, 2,3, 4,5,6}.$

$
Because$the$event$of$rolling$dice$is$independent$each$other,$so$that$the$second$rolling$dice$is$not$affected$by$the$result$from$the$
first$rolling$dice.$
The$event$“number$3”$of$the$rolling$dice$is$ {3}.$

Therefore,$we$can$obtain$the$probability$as$follows.$
n(E) 1
P (E) = = $$
n (S ) 6

19
16.$The$correct$answer$is$(A).$
We$use$binomial$distribution$to$solve$this$question.$
p = 0.05
q = 1 − p = 0.95
n!
p {3} = p x q n− x !! !
x !( n − x )!
10! 3 10 −3
p {3} = ( 0.05) ( 0.95)
3!(10 − 3)!
p {3} = 0.0105

20
Chapter 3

Computational
Tools

A. Spreadsheets
B. Programming
17.$Which$statement$is$correct$related$to$the$HLOOKUP()$function$in$the$Microsoft$Excel.$
(A)$to$search$for$a$value$in$the$top$row$of$a$table$or$an$array$of$values,$then$returns$a$value$in$the$same$column$from$a$row$in$the$
table$or$array$
(B)$to$search$for$a$value$in$the$column$of$a$table$or$an$array$of$values,$then$returns$a$value$in$the$same$row$from$a$column$in$the$
table$or$array$$
(C)$to$sort$of$data$in$a$table$or$an$array$by$ascending$order$
(D)$to$sort$of$data$in$a$table$or$an$array$by$descending$order$
$
$
$
$
$
18.$Several$lines$of$code$program$are$written$as$follows.$
$ a$=$1;$
$ b$=$1;$
$ while$a$<=$5$
$ $ b$=$b$+$2;$
$ $ a$=$a$+$1;$
$ do$
What$is$the$last$value$of$b$in$the$end$of$the$program?$
(A)$5$
(B)$7$
(C)$9$
(D)$11$
$

22
19.$How$many$cells$are$there$in$the$range$D2:F10$of$Microsoft$Excel$spreadsheet?$
(A)$16$
(B)$18$
(C)$24$
(D)$27$
$
$
$
$
$
20.$Determine$the$most$appropriate$function$to$create$looping$in$a$certain$number.$
(A)$while$…$do$
(B)$if$…$else$…$end$
(C)$for$…$end$
(D)$if$…$else$if$…$end$
$
$
$
$ $

23
$
$
$
$
$
$
$
$
$
$
$
$
$
$
$
$
$
$
$
$
$
$
$

24
17.$The$correct$answer$is$(A).$
$
$
$
$
$
18.$The$correct$answer$is$(D).$
The$looping$condition$can$be$solved$by$observing$the$variable$data$for$each$loop.$
Condition$ b$ a$
Initialize$ 1$ 1$
First$looping$a$=$1$ 3$ 2$
First$looping$a$=$2$ 5$ 3$
First$looping$a$=$3$ 7$ 4$
First$looping$a$=$4$ 9$ 5$
First$looping$a$=$5$ 11$ 6$
First$looping$a$=$6$ looping$stops$
$
We$can$solve$the$question$and$find$the$last$value$of$b$is$11.$
$
$
$
$
$

25
19.$The$correct$answer$is$(D).$
For$the$range$D2:J10,$we$have$3$columns$and$9$rows,$so$that$the$total$cells$are$27$cells.$
$
$
$
$
$
$
20.$The$correct$answer$is$(C).$
“if$…$else$…$end”$and$“if$…$else$if$…$end”$are$the$function$to$create$conditional$statements,$while$for$the$looping$function$we$need$
to$use$“while$…$do”$or$“for$…$end”.$If$we$know$the$certain$number$of$looping,$“for$…$end”$function$will$be$very$good$option.$
$ $

26
Chapter 4

Ethics and
Professional
Practice
A. Codes of Ethics
B. Professional Liability
C. Licences
D. Sustainability
E. Professionalism
F. Contracts
21.$Sam$graduated$from$the$university$with$a$bachelor$degree$in$Civil$Engineering.$He$also$passed$FE$exam$during$his$senior$year$in$
the$university$and$currently$works$as$an$Engineer$in$Training$(EIT)$for$a$local$engineering$firm$under$the$supervision$of$a$
licensed$professional$engineer,$Dom.$The$firm$is$about$to$join$a$tender$of$structural$design$on$new$apartment$building$in$the$
city.$In$the$other$hand,$Sam’s$father$is$the$investor$of$the$new$apartment$building.$What$should$Sam$do?$
(A)$Persuade$his$father$to$select$Sam’s$design$bid$as$the$winner$
(B)$Cancel$the$participation$of$tender$project$
(C)$Continue$to$make$the$best$design$related$to$the$design$requirement$
(D)$Hold$a$meeting$with$his$father$and$his$supervisor$to$develop$a$good$connection$between$them$
$
$
$
$
22.$As$a$fresh$graduate,$you$were$hired$to$be$a$junior$field$supervisor$in$a$new$hotel$project.$You$found$that$the$project$manager$
treated$laborer$in$an$unsafe$and$inhumane$manner,$but$it’s$legal$in$the$country.$You$reported$this$problem$to$the$company$
executives$about$this$problem,$but$you$were$told$that$they$should$do$it$in$order$to$be$competitive$with$time$and$cost.$You$were$
also$told$that$you$should$accept$the$culture$company$as$the$way$things$are.$What$should$you$do$to$satisfy$the$code$of$ethics?$
(A)$Report$the$company$to$the$authorities$
(B)$Keep$quiet$and$follow$the$company$culture$
(C)$Quit$from$the$job$
(D)$Organize$strike/demonstration$together$with$the$laborer$
$
$
$

28
23.$Which$one$is$the$benefit$of$doing$ethical$behavior?$
(A)$Higher$salary$
(B)$Better$reputation$
(C)$Better$feeling$about$yourself$
(D)$None$of$the$above$
$
$
$
$
$
24.$Sam$and$Harry$are$two$licensed$professional$engineers.$They$have$submitted$design$bid$for$a$very$big$project.$Sam$submitted$
higher$bid$than$Harry.$The$client,$however,$preferred$Sam’s$work$so$that$the$client$asked$Sam$to$beat$the$Harry’s$bid$and$Sam$
would$win$the$big$project.$What$should$Sam$do?$
$
(A)$Withdraw$his$bid$and$let$the$client$choose$Harry’s$bid$
(B)$Follow$the$client’s$request$and$beat$Harry’s$bid$
(C)$Bargain$the$amount$of$bid$with$the$client$
(D)$Keep$the$previous$bid$and$let$the$client$decide$
$
$
$
$
$

29
25.$You$are$a$city$engineer$in$charge$of$selecting$bids$for$a$big$infrastructure$project$in$the$city.$There$is$one$contractor$who$came$
with$high$bid$and$offered$limited$edition$of$Rolex$watch.$You$know$you$love$Rolex$watches$and$you$also$know$that$the$lowest$
bid$will$be$selected$as$the$winner$of$project.$What$should$you$do?$
(A)$Accept$the$Rolex$watch$and$accept$the$bid$
(B)$Accept$the$Rolex$watch$and$reject$the$bid$
(C)$Reject$the$Rolex$watch$and$accept$the$bid$
(D)$Reject$the$Rolex$watch$and$reject$the$bid$
$
$
$ $

30
$ $

31
21.$The$correct$answer$is$(C).$
$
$
22.$The$correct$answer$is$(C).$
The$company$hasn’t$broken$any$laws,$so$there$is$no$authority$to$report$to.$You$could$show$your$protest$to$the$company$because$
it$used$unethical$business$practices.$You$could$organize$a$strike/demonstration$together$with$laborer,$but$it$might$be$illegal$in$
the$country.$Therefore,$at$least$you$should$quit$from$the$job$as$your$personal$choice$to$keep$your$ethics.$
$
$
23.$The$correct$answer$is$(D).$
By$doing$ethical$behavior,$you$will$not$get$any$benefits$because$ethical$behavior$promises$nothing.$
$
$
24.$The$correct$answer$is$(D).$
It$would$be$unethical$for$Sam$to$submit$another$bid$and$beat$Harry’s$bid.$It$is$illegal$depend$on$the$regulatory$agency.$Sam$
should$keep$his$bid$and$let$the$client$decide.$
$
$
25.$The$correct$answer$is$(C).$
You$should$not$accept$any$kind$of$gifts$from$parties$expecting$special$consideration,$so$that$you$should$reject$the$Rolex$watch.$
As$long$as$the$contractor$satisfies$all$the$bid$requirement$and$follows$the$bidding$rules,$the$bid$could$be$accepted.$

32
Chapter 5

Engineering
Economics

A. Cash Flow
B. Costs (Incremental, Average, Sunk, Estimating)
C. Economic Analyses
D. Expected Value and Risk
26.$What$is$the$approximate$annual$effective$interest$rate$of$money$invested$at$3%$per$year$and$compounded$quarterly?$
(A)$1.034%$
(B)$2.034%$
(C)$3.034%$
(D)$4.034%$
$
$
$
$
$
27.$A$bank$advertisement$offers$an$interest$rate$of$5.2%$per$year$with$continuous$compounding.$Find$the$approximate$effective$
annual$interest$rate.$
(A)$4.337%$
(B)$5.337%$
(C)$6.337%$
(D)$7.337%$
$
$
$
$
$
$

34
28.$A$tower$crane$was$just$bought$for$$70,000.$The$life$span$of$the$crane$is$estimated$as$10$years$and$the$salvage$value$is$$5,800.$
Determine$the$approximate$value$of$depreciation$for$this$crane.$
(A)$$4,620$
(B)$$5,800$
(C)$$6,420$
(D)$$7,000$
$
$
$
$
$$
29.$A$table$manufacturing$factory$has$the$total$operating$cost$$250,000$per$year,$including$salaries,$rent$fee,$and$depreciation$cost.$
Each$table$needs$$42,$and$the$price$of$sale$is$$69.$Find$the$break>even$sales$volume$of$the$factory?(63)$
(A)$6250$tables/year$
(B)$7250$tables/year$
(C)$8250$tables/year$
(D)$9250$tables/year$
$
$
$
$
$

35
30.$A$home$company,$which$manufactures$bags,$costs$$25$for$the$material$and$equipment$each$bag$and$$6$for$labor$cost$each$bag.$If$
the$company$uses$machine$to$produce$bags,$it$will$cost$$500$for$the$material$and$equipment$each$bag$and$$1$for$the$operator$
cost$each$bag.$If$the$company$produces$65$bags$per$month,$how$long$will$the$company$reach$the$break>even>point?$
(A)$1.12$months$
(B)$1.23$months$
(C)$1.35$months$
(D)$1.46$months$
$
$
$
$
$
31.$The$maintenance$cost$of$an$investment$is$$15,000$per$year$for$the$first$5$years$and$$7,500$per$year$after$5$years.$Assume$that$
the$investment$will$never$end.$If$the$interest$rate$is$12%,$determine$the$approximate$present$worth$of$annual$disbursement.$
(A)$$26,936$
(B)$$27,936$
(C)$$28,936$
(D)$$29,936$
$
$ $

36
$ $

37
26.$The$correct$answer$is$(C).$
m
! r"
i = $1 + % − 1
& m'
4
! 0.03 "
i = $1 + % −1 $
& 4 '
i = 0.03034 = 3.034%
$
$
$
$
27.$The$correct$answer$is$(B).$
Continuous$compounding$can$be$approximated$by$using$large$number$for$ m ,$such$as$1000.$
$
m
! r"
i = $1 + % − 1
& m'
1000
! 0.052 "
i = $1 + % −1 $
& 1000 '
i = 0.05337 = 5.337%
$
$
$
$
$
$

38
28.$The$correct$answer$is$(C).$
The$depreciation$value$is$equal$to$the$difference$between$initial$cost$ C $and$salvage$value S n ,$then$divided$by$the$life$span$ n $.$

$
$
C − Sn
D=
n
$70, 000 − $5,800
D= $
10
D = $6, 420
$
$
$
$
29.$The$correct$answer$is$(D).$
The$break>even>point$occurs$when$costs$(which$are$negative)$and$sales$(which$are$positive)$total$zero.$Let$ n $be$the$number$of$
tables$per$year$at$break>even>point.$
$
$
−$250, 000 − n$42 + n$69 = 0
n$27 = $250, 000 $
n = 9259 ≈ 9250tables/year
$
$
$

39
30.$The$correct$answer$is$(D).$
Total$cost$for$each$bag$(hand>made)$=$ $25 + n$6 $where$n$is$the$number$of$bags$
$
Total$cost$for$each$bag$(machine$processed)$=$ $500 + n$1 $where$n$is$the$number$of$bags$
$
$25 + n$6 = $500 + n$1
n$5 = $475 $
n = 95bags
$
95
t= = 1.46months $$
65
$
31.$The$correct$answer$is$(B).$
The$costs$or$benefits$for$a$cash$flow$that$repeat$should$be$broken$into$different$benefits$and$costs$that$all$start$or$finish$at$the$
time$of$interest.$Take$$15,000$cost$that$repeats$for$5$years$and$break$it$into$two$$7,500$costs:$the$first$$7,500$cost$goes$on$5$
years$and$the$second$$7,500$cost$goes$on$for$infinitely.$
$
!P " !P "
PV = ( $7,500 ) $ ,12%,5 % + ($7,500 ) $ ,12%, ∞ %
&A ' &A '
PV = ( $7,500 )( 3.6048 ) + ( $7,500 )( 0.12 ) $
PV = $27,936
$$ $

40
Chapter 6

Statics

A. Resultants of Force Systems


B. Force Systems
C. Equilibrium of Rigid Bodies
D. Frames and Trusses
E. Centroids
F. Moments of Inertia
G. Static Friction
32.$Determine$the$length$of$vector$ V1 + V2 + V3 $where$the$vectors$are$written$as$follows.$

V1 = 5k m
V2 = 8i m $
V3 = 2 j m
(A)$8.77$m$
(B)$9.64$m$
(C)$9.90$m$
(D)$10.49$m$
$
$
33.$Find$the$moment$at$point$B$due$to$the$force$ P $as$shown$in$the$following$figure.$

P = 200 kips
60
¢X

5 ft
A
B
6 ft 6 ft
$
(A)$200$kips>ft$
(B)$233$kips>ft$
(C)$266$kips>ft$
(D)$300$kips>ft$
$
42
34.$Referring$to$the$previous$figure,$determine$the$vertical$reaction$at$support$A.$

( )
(A)$ 34.34 kips ↓ $

(B)$ 34.34 kips (↑ )$

(C)$ 44.34 kips (↓) $

(D)$$ 44.34 kips (↑) $

$
$
$
35.$Given$an$irregular$shape$as$shown$below$has$the$area$of$50$cm2$and$the$inertia$moment$about$X>axis$is$3654.8$cm4.$Determine$
the$inertia$moment$about$X’>axis.$

centroidal
axis Xc
4 cm
5 cm

X
X'
$
(A)$2854.8$cm4$
(B)$3104.8$cm4$
(C)$3854.8$cm4$
(D)$4104.8$cm4$
$

43
Question$36$and$37$are$based$on$this$following$figure.$Three$forces$are$connected$at$point$
$
$

300 N F1

3 1
5 O 1

F2 $
$
$
36.$Determine$the$required$force$ F1 $to$satisfy$the$force$equilibrium$at$point$O$is$zero.$

(A)$313.8$N$
(B)$363.8$N$
(C)$413.8$N$
(D)$463.8$N$
$
$
$
$

44
37.$$Determine$the$required$force$ F2 $to$satisfy$the$force$equilibrium$at$point$O$is$zero.$

(A)$311.6$N$
(B)$361.6$N$
(C)$411.6$N$
(D)$461.6$N$
$
$
38.$ Three$ ropes$ are$ connected$ each$ other$ to$ hold$ a$ box$ of$ 50$ N$ weight.$ Determine$ the$ tension$ T1 $and$ T2 $if$ the$ system$ is$ in$

equilibrium.$

T1

¢X
60
T2

70 N
$
(A)$ T1 = 40.41N , T2 = 80.83N $

(B)$ T1 = 80.83N , T2 = 40.41N $

(C)$ T1 = 50 N , T2 = 80.83N $

(D)$ T1 = 80.83N , T2 = 50 N $

45
39.$A$book$shelf$with$weight$of$300$N$is$supported$by$hinge$A$and$B$on$the$wall.$Find$the$horizontal$reaction$at$hinge$A.$

2.5 m
5m
B

3m

300 N $
(A)$ 140 N ( →) $

(B)$ 160 N ( →) $

(C)$ 180 N ( →) $

(D)$$ 200 N ( →) $

$
40.$Which$one$is$defined$by$the$centripetal$force$acting$on$a$particle$which$travels$in$a$circular$path$at$constant$velocity?$
1
(A)$ mω 2 r 2 $
2
(B)$ mω 2 r 2 $
1
(C)$ mω 2 r $
2
(D)$ mω 2 r $
$

46
$

47
32.$The$correct$answer$is$(B).$

V1 + V2 + V3 = V12 + V2 2 + V32
V1 + V2 + V3 = 52 + 82 + 22 $
V1 + V2 + V3 = 9.64 m
$
$
$
$
$
33.$The$correct$answer$is$(C).$
Take$point$B$as$the$center$of$rotation$and$assume$that$the$clockwise$moment$is$positive$moment.$ $
$
M B = ( P sin 60° ) 5 − ( P cos 60° ) 6
M B = ( 200 × sin 60° ) 5 − ( 200 × cos 60° ) 6 $
M B = 266.03 kips-ft ≈ 266 kips-ft
$
$
$
$
$
$
$
$

48
34.$The$correct$answer$is$(C).$
In$order$to$get$the$vertical$reaction$at$support$A,$we$need$to$check$the$equilibrium$at$point$B.$
Hinge$support$can’t$resist$moment,$so$that$the$equilibrium$moment$at$point$B$is$0.$
Assume$that$the$vertical$reaction$in$A$( VA $)$is$downward.$

∑M B =0
− (VA ) 6 + ( P sin 60° ) 5 − ( P cos 60° ) 6 = 0
$
−6VA + ( 200 × sin 60° ) 5 − ( 200 × cos 60° ) 6 = 0
VA = 44.34 kips ( ↓ )

$
$
$
$
35.$The$correct$answer$is$(D).$
We$need$to$find$the$inertia$moment$about$its$centroidal$axis$first,$then$we$can$solve$the$inertia$moment$about$other$axis.$
I x = I c + Ay 2
3654.8 = I c + 50 × 42 $$
I c = 2854.8cm4

I x ' = I c + Ay 2
I x ' = 2854.8 + 50 × 52 $
I x ' = 4104.8cm 4
$
$

49
36.$The$correct$answer$is$(B).$
Because$we$have$two$unknown$forces$in$vertical$(Y)$direction,$we$need$to$use$equilibrium$in$horizontal$(X)$direction$first.$
$

∑F x =0
! 5 " ! 1 "
−300 $
2 2
% + F1 $ 2 2 % = 0$
& 3 +5 ' & 1 +1 '
F1 = 363.8 N
$
$
$
$
$
37.$The$correct$answer$is$(C).$
From$ question$ 36,$ we$ can$ find$ one$ unknown$ force.$ Therefore,$ in$ this$ question$ we$ can$ check$ the$ equilibrium$ in$ vertical$ (Y)$
direction$to$find$the$other$unknown$force.$

∑F y =0
! 3 " ! 1 "
300 $
2 2
% + 363.8 $ 2 2 % − F2 = 0 $
& 3 +5 ' & 1 +1 '
F2 = 411.6 N
$
$
$
$

50
38.$The$correct$answer$is$(B).$

∑F y =0
T1 sin 60° − 70 = 0 $
T1 = 80.83N

∑F x =0
T1 cos 60° − T2 = 0 $
T2 = 40.41N
$
$
$
$
39.$The$correct$answer$is$(C).$
In$order$to$get$the$horizontal$reaction$at$support$A,$we$need$to$check$the$equilibrium$at$point$B.$
Hinge$support$can’t$resist$moment,$so$that$the$equilibrium$moment$at$point$B$is$0.$
Assume$that$the$horizontal$reaction$in$A$( H A $)$is$to$the$right.$

∑M B =0
− ( 300 )1.5 + ( H A ) 2.5 = 0 $
H A = 180 N ( → )
$
$
$
$

51
40.$The$correct$answer$is$(D).$
Fcp = macp
v2
Fcp = m
r
2 $$ $
Fcp =m
(ω r )
r
2
Fcp = mω r

52
Chapter 7

Dynamics

A. Kinematics (Particles & Rigid Bodies)


B. Mass Moments
C. Newton’s 2nd Law (Particles & Rigid Bodies)
D. Impulse and Momentum
E. Work and Energy (Particles & Rigid Bodies)
41.$A$stone$with$mass$150$gram$is$thrown$upward$with$velocity$15$m/s.$Find$the$kinetic$energy$of$stone$when$the$stone$is$2$m$
above$the$initial$point!$
(A)$13.635$Joule$
(B)$13.735$Joule$
(C)$13.835$Joule$
(D)$13.935$Joule$
$
$
$
$
42.$A$particle$moves$by$the$function$ v (t ) = 10t + 3$in$unit$of$m/s.$Calculate$the$distance$traveled$by$the$particle$from$3$to$7$seconds.$

(A)$40$m$
(B)$54$m$
(C)$212$m$
(D)$266$m$
$
$
$
$
$
$
$
$

54
43.$A$1500$kg$car$moves$with$a$velocity$of$60$km/hour,$then$the$car$enters$a$curve$with$radius$of$1$km.$Determine$the$minimum$
coefficient$of$friction$between$the$road$and$the$car$so$that$the$car$can$moves$in$the$curve$without$any$brakes.$
(A)$0.018$
(B)$0.023$
(C)$0.028$
(D)$0.033$
$
$
$
$
$
44.$An$8$kg$box$is$pushed$upward$following$the$inclined$plane$of$15°$to$the$horizontal$plane.$Determine$the$force$required$to$push$
the$box$until$it$accelerates$2$m/s2$at$the$inclined$plane.$Assume$that$the$coefficient$of$friction$between$the$inclined$plane$and$
box$is$0.3.$
(A)$49$N$
(B)$59$N$
(C)$69$N$
(D)$79$N$
$
$
$
$

55
45.$Assume$that$a$blue$billiard$is$located$in$the$left$side$of$a$red$billiard$ball.$The$1.5$kg$blue$billiard$ball$moves$to$the$right$with$a$
constant$velocity$of$30$m/s;$while$the$2.5$kg$red$billiard$ball$moves$to$the$left$with$a$constant$velocity$of$15$m/s.$Those$blue$and$
red$billiard$balls$collide,$after$that$they$move$together$to$the$right$with$the$same$velocity.$Find$the$velocity$of$both$billiard$balls$
after$collision.$
(A)$1.125$m/s$
(B)$1.375$m/s$
(C)$1.675$m/s$
(D)$1.875$m/s$
$
$
$
$
$$
46.$A$box$moves$and$travels$8$m$from$the$top$of$inclined$plane$to$the$bottom$of$incline$plane.$The$inclination$angle$of$the$inclined$
plane$is$20°$to$the$horizontal$plane.$Assume$that$the$coefficient$of$friction$between$the$inclined$angle$and$box$is$0.2.$If$the$box$
starts$from$rest$at$the$top$of$inclined$plane,$find$the$velocity$when$it$reaches$the$bottom$of$inclined$plane.$
(A)$2.9$m/s$
(B)$3.9$m/s$
(C)$4.9$m/s$
(D)$5.9$m/s$
$
$
$

56
$
$
$
$
$
$ $

57
41.$$The$correct$answer$is$(D).$
The$velocity$at$the$height$of$2$m$:$
Vt 2 = V0 2 − 2 gh
Vt 2 = 152 − 2 × 9.8 × 2
$
Vt 2 = 185.8
Vt = 185.8 = 13.631m / s
The$kinetic$energy$at$the$height$of$2$m$:$
1 1
Ek = mV 2 = × 0.15 ×185.8 = 13.935 J $
2 2
$
$
$
$
42.$The$correct$answer$is$(C).$
Because$the$velocity$function$is$in$terms$of$time,$the$distance$equation$can$be$written$as$follows.$
$

s ( t ) = ∫ v ( t ) dt
7
s ( t ) = ∫ (10t + 3) dt
3
7
s ( t ) = 5t 2 + 3t $$
3

( ) (
s ( t ) = 5 × 7 2 + 3 × 7 − 5 × 32 + 3 × 3 )
s ( t ) = 212m

58
43.$The$correct$answer$is$(C).$
$
Ff = Fcp
v2
µ mg = m
r
2
$$
µ × 9.8 =
( 60 ×1000 / 3600 )
1000
µ = 0.028
$
$
$
$
$
44.$The$correct$answer$is$(B).$
We$need$to$define$the$force$worked$to$the$moving$box$in$the$inclined$plane,$we$will$obtain$the$force$required$to$be$applied$to$
the$moving$box.$
$

∑ F = ma
F − mg sin15° − µ mg cos15° = ma $
F − 8 × 9.8sin15° − 0.3 × 8 × 9.8cos15° = 8 × 2
F = 59 N
$
$
$

59
45.$The$correct$answer$is$(D).$
Assume$that$right$direction$is$positive$and$left$direction$is$negative.$We$use$the$law$of$conservation$of$momentum$as$follows.$
$
pinitial = pend
( mv )blue + ( mv )red = ( mblue + mred ) v $
1.5 × 30 + 2.5 × ( −15 ) = (1.5 + 2.5 ) v
v = 1.875m / s
$
$
$
$
46.$The$correct$answer$is$(C).$
First,$we$need$to$define$the$force$worked$to$the$moving$box$in$the$inclined$plane,$we$will$obtain$the$acceleration$of$the$moving$
box.$

∑ F = ma
mg sin 20° − µ mg cos 20° = ma
$$
9.8sin 20° − 0.2 × 9.8cos 20° = a
a = 1.5m / s 2
$
After$the$acceleration$is$known,$we$use$the$principal$of$motion$particles$to$find$the$final$velocity$of$moving$box.$
Vt 2 = V0 2 + 2as
Vt 2 = 02 + 2 × 1.5 × 8 $
Vt = 24 = 4.9m / s
$

60
Chapter 8

Mechanics of
Materials

A. Shear and Moments


B. Stress (Axial, Bending, Torsional, Shear)
C. Deformations (Axial, Bending, Torsional, Shear)
D. Combined Loads
E. Principal Loads
F. Mohr’s Circle
G. Columns (Buckling, Boundaries)
H. Composite Sections
I. Deformations (Plastic, Elastic)
J. Stress - Strain
Question$47$and$48$are$based$on$the$following$figure.$
$
$
100 N/m

200 N

5m
$
$
$
$
47.$Determine$the$vertical$reaction$of$the$cantilever$beam.$

( )
(A)$ 50N ↑ $

(B)$ 50N (↑ ) $

(C)$ 100N (↑ ) $

(D)$ 100N (↓ ) $

$
$
$
$
$$

62
48.$Determine$the$moment$at$the$fixed$end$support$of$the$cantilever$beam.$
(A)$583.33$Nm$(clockwise)$
(B)$583.33$Nm$(counter$clockwise)$
(C)$375$Nm$(clockwise)$
(D)$375$Nm$(counter$clockwise)$
$
$
$
$
$
49.$A$steel$rebar$is$pulled$in$tension$until$it$passes$its$yield$point$and$it$has$a$little$permanent$deformation.$After$that,$the$rebar$is$
unloaded.$Which$of$these$following$statements$is$true$if$the$rebar$is$pulled$again?$
(A)$The$ultimate$strength$will$decrease$
(B)$The$yield$point$remains$the$same$
(C)$The$ductility$will$increase$
(D)$The$yield$point$will$change$
$
$
$
$
$
$
$

63
50.$$Find$the$correct$equation$to$define$normal$strain$ ε .$Assume$that$the$symbol$ δ $is$elongation$and$ L $is$the$length$of$specimen.$
L
(A)$ ε = $$
δ
L +δ
(B)$ ε = $
L
δ
(C)$ ε = $
L +δ
δ
(D)$ ε = $
L
$
$
$
$
51.$$A$steel$rod$is$stressed$by$a$tension$force$of$250$N.$It$is$found$that$the$rod$has$length$of$45$m$and$diameter$of$1.5$mm.$If$the$
modulus$elasticity$of$steel$rod$is$assumed$as$ 2 ×105 MPa ,$determine$the$strain$of$the$steel$rod$due$to$the$applied$force.$
(A)$ 5.16 ×10−4 $
(B)$ 6.57 ×10−4 $
(C)$ 7.07 ×10−4 $
(D)$ 8.24 ×10−4 $
$
$
$
$
$

64
52.$Find$the$normal$stress$of$cantilever$beam$AB$shown$in$the$following$figure.$
$

0.3 m
100 N 250 N
5m
0.1

0.6 m 0.6 m

A B C
$
$
(A)$3333$N/m2$(compression)$
(B)$5555$N/m2$(compression)$
(C)$3333$N/m2$(tension)$
(D)$5555$N/m2$(tension)$
$
$
$
$
$
$
$
$
$

65
53.$Find$the$maximum$shear$stress$of$this$element.$
$
50 MPa

50 MPa
120 MPa 120 MPa

50 MPa

50 MPa
$
(A)$46$MPa$
(B)$61$MPa$
(C)$72$MPa$
(D)$85$MPa$
$
$
$
$
$
$
$
$

66
54.$A$2$m$hollow$cylindrical$rod$has$the$maximum$allowable$shear$stress$40$MPa$and$the$maximum$allowable$twist$angle$0.025$
radians.$Find$the$maximum$allowable$torque$ T $to$this$hollow,$considering$the$following$material$properties$and$the$maximum$
allowable$stress.$
G = 70000MPa
J = 250cm 4 $
$
$
T

7.5 cm
5 cm
T
2m
$
$
$
(A)$2.19$kNm$
(B)$3.25$kNm$
(C)$4$kNm$
(D)$5.15$kNm$
$
$
$
$
$

67
55.$The$stress>strain$relationship$of$steel$reinforcement$is$shown$in$the$following$figure.$Determine$the$strain$hardening$part.$
$
$
fs
D
E
B C

A £s̀
$
(A)$A>B$
(B)$B>C$
(C)$C>D$
(D)$D>E$
$
$
56.$Determine$the$location$of$maximum$deflection$of$a$simple>supported$beam$loaded$by$distributed$loading.$
(A)$at$the$support$
(B)$at$the$ 1 4 L $from$the$support$
(C)$at$the$middle$of$span$
(D)$at$the$ 3 4 L $from$the$support$
$

68
57.$Determine$the$maximum$principal$strain$at$a$point$where$ ε x = 1000µ m ,$ ε y = −600µ m ,$and$ ε xy = 800 µ m .$

(A)$ 695 µ m $
(B)$ 895 µ m $
(C)$ 1095 µ m $
(D)$ 1295 µ m $
$
$
$
$
$
$
$
$
$
$
$
$
$
$
$
$
$
$

69
Question$58$and$59$are$based$on$this$simple>supported$beam$below.$
$
5 kN

3m 3m
$
$
$
58.$Solve$which$of$the$following$figures$is$the$correct$shear$force$diagram.$

2.5 kN +

- 2.5 kN
(A)$ $

+ 2.5 kN

2.5 kN -
(B)$ $

2.5 kN +
-
2.5 kN
(C)$ $

+ 2.5 kN
-
2.5 kN
(D)$ $
$

70
59.$Solve$which$of$the$following$figures$is$the$correct$bending$moment$diagram.$

(A)$ 7.5 kNm $

(B)$ 15 kNm $
15 kNm

+
(C)$ $
7.5 kNm

+
(D)$ $
$
$ $

71
$
$
$
$
$
$
$
$
$
$
$
$
$
$
$
$
$
$
$
$
$
$
$

72
47.$The$correct$answer$is$(A).$
1
V = × 5 ×100 − 200
2 $
V = 50 N ( ↑ )

$
$
$
$
48.$The$correct$answer$is$(A).$
Assume$that$the$clockwise$moment$is$positive$moment.$
$
!1 "! 5 "
M + % × 5 × 100 & % & − ( 200 )(5 ) = 0
'2 (' 3 ( $
M = 583.33 Nm
$
$
49.$The$correct$answer$is$(D).$
Due$to$the$permanent$deformation,$the$yield$point$will$change$into$a$new$point.$You$can$confirm$this$by$considering$the$stress>
strain$relationship$of$steel$rebar.$
$
$
$
$
$

73
50.$The$correct$answer$is$(D).$
Strain$is$defined$by$elongation$per$length.$
$
$
$
$
51.$The$correct$answer$is$(C).$
$
PL
δ=
EA
250 × 45000
δ=
(
2 ×10 × 0.25 × π × 1.52
5
)
δ = 31.83mm $$
δ
ε=
L
31.83
ε=
45000
ε = 7.07 ×10−4
$
$
$
$
$
$
$

74
52.$The$correct$answer$is$(A).$
Before$we$find$the$normal$stress$ σ AB ,$we$need$to$find$the$force$of$element$AB$( FAB )$first.$

$
From$the$free>body$diagram,$we$can$find$ FAB = −150 N .$

$
FAB
σ =
AB
AAB
−150
σ = $
AB
0.15 × 0.3
σ AB
= −3333 N / m 2

$
$
$
$
$
$
$
$
$
$
$
$
$

75
$
53.$The$correct$answer$is$(B).$
From$the$stress$condition$of$element,$we$can$draw$Mohr$circle$as$below.$
$
$

τmax
(-120,50)

σ2 σavg σ1 σ

(-50,-50)

$
$
$
−120 − 50
σ avg = = −85MPa
2 $$
2 2
τ max = ( −120 + 85) + (50) = 61.03MPa

$
$
$
$
$

76
$
54.$The$correct$answer$is$(A).$
We$check$both$limitation$of$maximum$allowable$shear$stress$and$maximum$allowable$twist$angle,$and$take$the$minimum$value$
as$the$maximum$allowable$torque.$
$
Tro
τ max =
J
T × 25
40 = $$
2500000
T = 4 ×106 Nmm = 4kNm
$
TL
φmax =
GJ
T × 2000
0.025 = $
70000 × 2500000
T = 2187500 Nmm = 2.1875kNm
$
$
We$take$the$maximum$allowable$torque$as$2.1875$kNm.$
$
$
$
$
$
$

77
$
55.$The$correct$answer$is$(C).$
$
$
$
56.$The$correct$answer$is$(C).$
The$maximum$deflection$is$occurred$at$the$location$of$maximum$moment,$which$for$simple>supported$structure$with$
distributed$loading,$the$maximum$moment$occurs$in$the$middle$of$span.$
$
$
$
57.$The$correct$answer$is$(C).$
$
2 2
εx +εy # ε x − ε y $ # ε xy $
ε max,min = ± % & +% &
2 ' 2 ( ' 2 (
2 2
1000 − 600 # 1000 + 600 $ # 800 $
ε max,min = ± % & +% &
2 ' 2 ( ' 2 (
ε max,min = 200 ± 894.43 µ m $
ε max = 1094.43 µ m
ε min = −694.43 µ m
$
$
$

78
$
58.$The$correct$answer$is$(A).$
To$be$able$to$draw$shear$force$diagram,$we$need$to$calculate$the$reaction$at$support.$Because$the$loading$is$applied$at$the$
middle$of$span,$the$reaction$at$both$supports$is$half$of$applied$loading.$In$this$case,$the$reaction$is$2.5$kN.$
$
$
$
$
59.$The$correct$answer$is$(D).$
The$bending$moment$for$this$case$is$linear$increase$until$the$maximum$moment$is$located$when$the$shear$force$diagram$is$zero.$

79
Chapter 9

Materials

A. Mixes (Concrete, Asphalt)


B. Testing Methods
C. Physical & Mechanical Properties
60.$Based$on$test$result,$it$is$found$that$the$stress$of$a$new$material$is$320$MPa$and$the$strain$is$4.5%$at$the$elastic$limit.$Determine$
the$elastic$modulus$of$this$material$based$on$that$data.$
(A)$5000$MPa$
(B)$6000$MPa$
(C)$7000$MPa$
(D)$8000$MPa$
$
$
$
$
61.$ASTM$(American$Standard$Testing$and$Material)$provides$a$lot$of$standard$testing$method$for$materials.$Which$of$the$following$
testing$methods$is$included$for$aggregates?$
(A)$Sieve$analysis$
(B)$Relative$density$
(C)$Absorption$
(D)$All$of$the$above$are$correct$
$
$
$
$
$
$

81
62.$A$steel$rebar$with$elastic$modulus$of$200000$MPa$is$subjected$to$compressive$force$and$the$rebar$has$a$strain$of$0.0065.$If$the$
diameter$of$steel$rebar$is$9.53$mm,$find$the$applied$compressive$force.$
(A)$90730$N$
(B)$91730$N$
(C)$92730$N$
(D)$93730$N$
$
$
$
$
63.$Which$of$the$following$chemical$admixtures$is$used$to$increase$the$workability$of$concrete?$
(A)$Retarders$
(B)$Accelerators$
(C)$Plasticizers/water$reducers$
(D)$Air$entraining$agents$
$
$
$
$
$
$
$
$

82
64.$Marshall$test$is$well>known$stability$test$for$….$
(A)$Concrete$
(B)$Steel$
(C)$Wood$
(D)$Asphalt$
$
$
$
$
$
$
$
$
$
$
$
$
$
$
$
$
$
$

83
$
$ $

84
60.$The$correct$answer$is$(C).$
$
σ
E=
ε
320
E= $$
0.045
E = 7111MPa ≈ 7000MPa
$
$
$
$
$
61.$The$correct$answer$is$(D).$
C136>06$covers$the$standard$test$method$for$sieve$analysis$for$fine$and$coarse$aggregates.$
C127>12$covers$the$standard$test$method$for$density,$relative$density$(specific$gravity),$and$absorption$of$coarse$aggregates.$
C128>12$covers$the$standard$test$method$for$density,$relative$density$(specific$gravity),$and$absorption$of$fine$aggregates.$
$
$
$
$
$
$
$
$

85
62.$The$correct$answer$is$(C).$
σ = Eε
σ = 200000 × 0.0065 $$
σ = 1300 MPa
$
F =σ A
#1 $
F = 1300 × & π × 9.532 ' $$
(4 )
F = 92729.7 N ≈ 92730 N
$
$
$
63.$The$correct$answer$is$(C).$
Plasticizer/water$reducer$is$used$to$increase$the$workability$of$concrete,$allowing$the$concrete$be$placed$more$easily.$
$
$
$
64.$The$correct$answer$is$(D).$
$ $

86
Chapter 10

Fluid
Mechanics

A. Flow Measurement
B. Properties of Fluids
C. Statics of Fluids
D. Equations (Impulse, Momentum, Energy)
65.$Which$of$the$following$options$is$component$of$motion$of$fluid?$
(A)$Rotation$
(B)$Translation$
(C)$Angular$distortion$
(D)$All$of$the$above$
$
$
$
$
66.$Assume$that$the$atmospheric$pressure$in$this$question$is$103500$Pa.$Determine$the$pressure$in$the$droplet$with$diameter$of$3.25$
mm$if$the$surface$tension$of$water$in$air$is$0.0828$N/m.$
(A)$103550$Pa$
(B)$103600$Pa$
(C)$103650$Pa$
(D)$103700$Pa$
$
$
$
$
$
$
$
$

88
67.$Find$the$gage$pressure$at$depth$1$m$from$the$bottom$of$water$tank$with$total$depth$is$3$m.$Assume$the$water$tank$is$contained$of$
water$with$ ρ = 1000kg / m3 .$
(A)$9.8$kPa$
(B)$19.6$kPa$
(C)$29.4$kPa$
(D)$39.2$kPa$
$
$
$
$$
68.$A$continuous$pipe$has$two$different$diameter$sizes$at$the$beginning$and$the$end$of$pipe.$The$test$result$shows$that$the$flow$
velocity$at$the$beginning$of$pipe$is$0.38$m/s.$The$diameter$at$the$beginning$of$pipe$is$1.5$times$larger$than$the$diameter$at$the$
end$of$pipe.$Determine$the$flow$velocity$at$the$end$of$pipe.$
(A)$0.168$m/s$
(B)$0.38$m/s$
(C)$0.855$m/s$
(D)$1.055$m/s$
$
$
$
$
$
$$

89
69.$A$turbine$with$flow$rate$of$0.22$m3/minute$has$the$pressure$drop$of$210$kPa.$Determine$the$power$output$of$the$turbine.$
(A)$0.46$kW$
(B)$0.58$kW$
(C)$0.63$kW$
(D)$0.77$kW$
$
$
$
70.$Based$on$which$concept$is$the$Bernoulli’s$law$for$fluid$flow?$
(A)$Continuity$equation$
(B)$Principle$of$conservation$of$momentum$
(C)$Principle$of$conservation$of$mass$
(D)$Principle$of$conservation$of$energy$
$
$ $

90
$ $

91
65.$The$correct$answer$is$(D).$
Fluid$has$three$components$of$its$motion:$rotation,$translation,$and$angular$distortion.$
$
$
$
$
66.$The$correct$answer$is$(B).$
$
Δp = pin − pout

= pin − pout
r $
2 × 0.0828
= pin − 103500
0.5 × 0.00325
pin = 103601.9 Pa ≈ 103600 Pa
$
$
$
67.$The$correct$answer$is$(B).$
Remember$that$the$pressure$is$calculated$based$on$the$depth$from$the$water$surface.$
$
p = ρ gh
p = 1000 × 9.8 × ( 3 − 1)
p = 19600 Pa = 19.6kPa $
$

92
68.$The$correct$answer$is$(C).$
By$using$the$continuity$law,$
ρ A1v1 = ρ A2v2
#1 2$ #1 2$
% π d1 & v1 = % π d 2 & v2
'4 ( '4 ( $
2
(1.5d2 ) × 0.38 = d 2 2 × v2
v2 = 0.855m / s
$
$
$
69.$The$correct$answer$is$(D).$
! 0.22 "
P = 210 × $ %
& 60 ' $$
P = 0.77 kW
$
$
$
$
70.$The$correct$answer$is$(D).$ $

93
Chapter 11

Hydraulics &
Hydrologics

A. Hydrology Basics
B. Hydraulics Basics
C. Systems of Pumping
D. Distribution Systems
E. Reservoirs
F. Groundwater Systems
G. Storm Sewer Design
71.$A$property$of$waste$water$system$has$length$of$280$m$and$width$of$150$m.$Assume$that$the$runoff$coefficient$of$the$property$is$
0.32.$Determine$the$discharge$if$the$intensity$of$rainfall$is$127$mm/hour.$
(A)$1500$m3/hour$
(B)$1600$m3/hour$
(C)$1700$m3/hour$
(D)$1800$m3/hour$
$
$
$
$
$
$
$
$
$
$
$
$
$
$
$
$
$

95
Question$72$and$73$are$based$on$the$following$information.$
Pumping$system$transfers$water$from$a$lake$to$a$water$tank$with$a$pipeline$of$250$m$length.$The$head$of$pipe$inlet$is$180$m,$while$
the$head$of$pipe$outlet$in$the$water$tank$is$195$m.$The$pipeline$is$made$of$cast$iron$(specific$roughness$is$taken$as$0.23$mm),$the$
diameter$of$which$is$35$cm.$
Assume$that$minor$losses,$entrances$losses,$and$exit$losses$in$the$pumping$system$can$be$ignored.$Consider$the$pump$efficiency$is$
75%.$ Also,$ assume$ that$ the$ water$ flow$ is$ steady$ and$ incompressible$ flow.$ The$ flow$ rate$ of$ water$ is$ 1.34$ m3/s.$ The$ kinematic$
viscosity$of$water$is$10>6$m2/s.$
$
$
72.$Determine$the$head$loss$in$the$piping$by$using$Darcy$equation.$
(A)$105$m$
(B)$155$m$
(C)$205$m$
(D)$255$m$$
$
$
73.$Determine$the$required$energy$to$pump$the$water$from$the$lake$to$water$tank.$
(A)$2.2$MW$
(B)$2.7$MW$
(C)$3.4$MW$
(D)$3.9$MW$
$
$

96
74.$Which$of$these$flows$has$the$flow$cross$section$that$does$not$vary$with$time$at$any$location$along$an$open$channel?$
(A)$Critical$flow$
(B)$Uniform$flow$
(C)$Steady$flow$
(D)$Non>uniform$flow$
$
$
$
75.$A$pumping$system$is$able$to$discharge$800$kPa$water$into$a$pipeline$with$length$of$50$m$and$diameter$of$8$cm.$The$initial$
velocity$Find$the$maximum$elevation$the$water$can$reach$if$we$neglect$the$friction.$
(A)$60$m$
(B)$70$m$
(C)$80$m$
(D)$90$m$
$
$
$
$
$
$
$
$
$

97
Question$76$and$77$are$based$on$the$following$information.$
A$large$complex$of$apartment$has$a$population$of$8000$people.$The$average$sewage$flow$for$this$complex$of$apartment$is$5000$
m3/day.$
$
76.$Determine$the$approximate$minimum$sewage$flow$for$this$complex$of$apartment.$
(A)$1415$m3/day$
(B)$1515$m3/day$
(C)$1615$m3/day$
(D)$1715$m3/day$
$
$
$
$
77.$Determine$the$approximate$peak$sewage$flow$for$this$complex$of$apartment.$
(A)$13250$m3/day$
(B)$14250$m3/day$
(C)$15250$m3/day$
(D)$16250$m3/day$
$
$
$
$

98
78.$A$sanitary$sewer$has$a$length$of$80$m$and$a$pipe$with$diameter$of$90$cm.$The$inlet$elevation$of$the$sewer$is$higher$1.2$m$than$the$
outlet$elevation.$Assume$that$the$Manning$roughness$coefficient$is$0.01,$constant$with$depth$of$flow.$Determine$the$sewer$
capacity$during$heavy$rainfalls$if$the$sewer$is$full$of$water$flow$with$no$surcharge.$
(A)$2.88$m3/s$
(B)$3.18$m3/s$
(C)$3.48$m3/s$
(D)$3.78$m3/s$
$
$ $

99
$ $

100
71.$The$correct$answer$is$(C).$
Q = CiA
Q = 0.32 × 0.127 × ( 280 ×150 ) $
Q = 1707m3 / hour ≈ 1700m3 / hour
$
$
$
72.$The$correct$answer$is$(C).$
Specific$roughness$of$cast$iron$=$ e = 0.23mm $
$
e 0.23
Relative$roughness$=$ = = 0.000657
D 350 $
$
Reynolds$number$is$determined$by$using$the$following$equation.$
VD
Re =
v
Q *D
Re = A
$ v $$
1.34 × 0.35
Re =
(0.25π × 0.352 )
10−6
Re = 4.875×106
$
$

101
After$getting$the$relative$roughness$and$Reynolds$number,$we$could$determine$the$friction$factor$from$Moody$diagram$shown$in$
the$below.$
!
!
!
!
!
!
!
!
!
!
!
!
!
!
!
!
!
!
!
!
!

102
The$friction$factor$is$ f = 0.0185 .$
Based$on$the$Darcy’s$equation,$the$head$loss$in$the$piping$is:$
2
" L #" V #
hf = f $ % $ %
& D ' & 2g '
2
"" # #
$ $1.34 % %
$
" 250 # $ & (
0.25π × 0.352 ) %
'
%
%$
h f = 0.0185 $ %$
& 0.35 '$ 2 × 9.8 %
$ %
$ %
& '
h f = 204.35m ≈ 205m

$
73.$The$correct$answer$is$(D).$
From$the$solution$number$72,$we$obtained$the$head$loss$ h f = 205m .$

Required$total$head$to$pump$the$water$is:$
h = ( zwatertank − zlake ) + h f
h = (195 − 180 ) + 205 $$
h = 220m
The$required$energy$with$efficiency$η = 75% $is:$
Q ρ w gh
W=
η
1.34 × 1000 × 9.8 × 220
W= $$
0.75
W = 3.852 × 106 W ≈ 3.9 MW
$

103
74.$The$correct$answer$is$(B).$
Uniform$flow$:$the$flow$cross$section$does$not$vary$with$time$at$any$location$along$an$open$channel.$
Steady$flow$:$the$flow$quantity$does$not$vary$with$time$at$any$location$along$an$open$channel.$
$
$
$
75.$The$correct$answer$is$(C).$
By$using$the$principal$of$conservation$energy:$
p1 v12 p2 v2
+ z1 + = + z2 + 2
γ 2g γ 2g
800000 42 0 42
+0+ = + z2 + $$
1000 × 9.8 2 × 9.8 1000 × 9.8 2 × 9.8
z2 = 81.6m ≈ 80m
$
$
$
$
$
$
$
$
$
$
$

104
76.$The$correct$answer$is$(B).$

$
From$the$sewage$flow$ratio$curves$above,$we$can$find$the$ratio$of$minimum>to>average$daily$sewage$flow$for$population$of$8000$
people$follows$curve$A2:$
Qmin P 0.2
=
Qavg 5
Qmin 80.2
= $$
5000 5
Qmin
= 0.303
5000
Qmin = 1515.7m3 / day ≈ 1515m3 / day
$
105
77.$The$correct$answer$is$(C).$

$
From$ the$ sewage$ flow$ ratio$ curves$ above,$ we$ can$ find$ the$ ratio$ of$ peak>to>average$ daily$ sewage$ flow$ for$ population$ of$ 8000$
people$follows$curve$G:$
Q peak 18 + P
=
Qavg 4+ P
Q peak 18 + 8
=
5000 4+ 8 $$
Q peak
= 3.05
5000
Q peak = 15251.3m3 / day ≈ 15250m3 / day

106
78.$The$correct$answer$is$(A).$
Slope$of$the$sewer$is:$
zinlet − zoutlet
S=
L
1.2
S= $$
80
S = 0.015
$
Because$the$sewer$is$full$of$water$flow$during$heavy$rainfall,$it$can$be$assumed$that$the$wetted$perimeter$is$equal$to$the$pipe$
perimeter$ P .$Therefore,$the$hydraulic$radius$is:$
A
R=
P
1
π × 0.92
R= 4 $$
π × 0.9
R = 0.225m
From$the$Manning$equation,$the$flow$velocity$is:$
1 2 3 12
v= R S
n
1 2 1
v= 0.225 3 0.015 2 $$
0.01
v = 4.53m / s
The$flow$capacity$is:$
Q = vA
"1 #
Q = 4.53 × % π × 0.92 & $$
'4 (
3
Q = 2.88m / s

107
Chapter 12

Structural
Analysis

A. Statically Determinant Force Analysis


B. Statically Determinant Deflection
C. Structural Determinancy & Stability
D. Load Paths
E. Indeterminate Structure Basics
79.$Determine$the$centroidal$coordinate$of$this$T>beam$shown$in$the$following$figure.$
50 cm

10 cm
35 cm
Y

O X
20 cm
$
(A)$(25$cm,$17.5$cm)$
(B)$(25$cm,$20$cm)$
(C)$(25$cm,$21.25$cm)$
(D)$(25$cm,$22.5$cm)$
$
$
$
$
$
$
$
$
$
$

109
Question$80>82$are$based$on$the$following$figure.$
15 kN

C E

4m
B
A
D F

3m 3m 3m
$
$
$
80.$Find$the$vertical$reaction$at$point$A$and$B.$

(A)$ VA = 7.5kN ↑ ( ) ; V = 7.5kN (↑) $$


B

(B)$ VA = 10kN (↑) ; V = 5kN (↑)$


B

(C)$ VA = 5kN (↑) ; V = 10kN (↑)$


B

(D)$ VA = 6kN (↑) ; V = 9kN (↑) $


B

$
$
$
$
$

110
81.$Determine$the$magnitude$of$compressive$force$in$member$AC.$
(A)$5.33$kN$
(B)$6.25$kN$
(C)$7.25$kN$
(D)$8.33$kN$
$
$
$
$
$
82.$Determine$the$force$and$magnitude$in$member$DF.$
(A)$3.75$kN$(tension)$
(B)$3.75$kN$(compression)$
(C)$6.67$kN$(tension)$
(D)$6.67$kN$(compression)$
$ $
$
$
$ $

111
$
$
$
$
$
$
$
$
$
$
$
$
$
$
$
$
$
$
$
$
$
$
$

112
79.$The$correct$answer$is$(C).$
The$T>beam$is$divided$into$two$areas$to$simplify$the$calculation.$

$
Because$the$section$is$symmetrical$about$vertical,$the$centroidal$X$is$in$the$center$ xc = 25cm $from$point$O$as$the$origin.$

yc =
∑Ay i i

∑A i

yc =
(50 ×10 ) 30 + ( 20 × 25)12.5 $
(50 ×10 ) + ( 20 × 25)
yc = 21.25cm
$
80.$The$correct$answer$is$(C).$

∑M A =0
15 × 6 − VB × 9 = 0 $$
VB = 10kN ( ↑ )

∑F y =0
VA + VB − 15 = 0
$$
VA + 10 − 15 = 0
VA = 5kN ( ↑ )

113
81.$The$correct$answer$is$(B).$
From$the$solution$number$80,$we$found$that$ VA = 6kN ,$so$that$we$can$draw$free>body$diagram:$
C

FAC

A
FAD D

VA= 5 kN
$
$
Check$the$equilibrium$of$Y$direction$at$point$A.$

∑F y =0
4
VA − FAC =0
5 $
4
5 − FAC = 0
5
FAC = 6.25kN ( compression )

$
$
$
$
$
$

114
82.$The$correct$answer$is$(A).$
Continuing$the$solution$from$number$81,$we$should$find$the$force$in$AD$before$calculating$the$force$in$DF.$We$check$the$
equilibrium$of$X$direction$at$point$A.$

C ∑F x =0
3
FAD − FAC =0
5 $
FAC
3
FAD − 6.25 × = 0
5
A FAD = 3.75kN ( tension )
FAD D

$
$ VA= 5 kN
$
$
$
After$the$force$in$AD$is$obtained,$we$can$check$the$equilibrium$of$X$direction$at$point$D.$
$$
C
∑F x =0
FAD − FDF = 0
$
3.75 − FDF = 0
FDF = 3.75kN (tension )
FAD FDF
$ A $
D F

115
Chapter 13

Structural
Design

A. Steel Component Design


B. Reinforced Concrete
83.$Based$on$ACI$318,$determine$the$top$flange$effective$width$of$this$T>beam.$

10 cm
35 cm
20 cm

400 cm
$
(A)$20$cm$
(B)$105$cm$
(C)$180$cm$
(D)$400$cm$
$
84.$Consider$the$rectangular$beam$with$the$dimension$in$question$83.$Assume$that$the$compressive$strength$of$concrete$is$21$MPa$
and$the$yield$strength$of$shear$reinforcement$is$280$MPa.$Determine$the$maximum$spacing$of$shear$reinforcement$(No.$#3$or$
diameter$of$9.53$mm)$to$satisfy$ultimate$shear$force$73$kN.$Neglect$any$axial$force$for$this$case.$Assume$the$effective$depth$of$beam$
is$0.8h.$
(A)$100$mm$
(B)$150$mm$
(C)$200$mm$
(D)$250$mm$
$
$

117
85.$Find$the$correct$condition$for$under>reinforced$failure$mechanism$in$reinforced$concrete.$
(A)$ As = As ,bal $$

(B)$ As < As ,bal $

(C)$ As > As ,bal $

(D)$None$of$the$above$
$
$
$
$
86.$Based$on$ACI$standard,$find$the$strength$reduction$factor$of$reinforced$concrete$members$for$shear;$tension;$compression$
without$spiral$reinforcement;$torsion$(write$it$by$order).$
(A)$0.75;$0.9;$0.75;$0.65$
(B)$0.9;$0.65;$0.75;$0.75$
(C)$0.65;$0.75;$0.9;$0.75$
(D)$0.75;$0.9;$0.65;$0.75$
$
$ $

118
$ $

119
83.$The$correct$answer$is$(B).$
!1 1
# 4 L = 4 × 420 = 105cm
#
b f = min $bw + 16ts = 20 + 16 ×10 = 180cm $
#center-to-center beam span = 400cm
#
%
$
84.$The$correct$answer$is$(C).$
The$shear$contribution$of$concrete:$

Vc = 0.17 f c' bd
Vc = 0.17 21 × 200 × ( 0.8 × 350 ) $$
Vc = 43626.12 N ≈ 43.6 kN
$
The$nominal$shear$strength$should$be$equal$or$larger$than$the$ultimate$shear$force$ (φ = 0.75) .$

φVc + φVs = Vu
0.75 × 43.6 + 0.75Vs = 73 $$
Vs = 53.733kN
The$required$spacing$of$shear$reinforcement:$
Av f ys d
Vs =
s
" 1 2#
% 2 × π × 9.53 & × 280 × ( 0.8 × 350 )
4
53733 = ' ( $
s
s = 208mm ≈ 200mm
$

120
85.$The$correct$answer$is$(B).$
Balanced$failure$occurs$when$ As = As ,bal ,$under>reinforced$failure$occurs$when$ As < As ,bal ,$over>reinforced$failure$occurs$when$

As > As ,bal .$

$
$
$
$
86.$The$correct$answer$is$(D).$ $

121
Chapter 14

Geotechnical
Engineering

A. Geology
B. Soil Classifications
C. Air-Water-Solid Phases
D. Lab & Field Tests
E. Effective Stresses
F. Retaining Walls
G. Shearing
H. Bearing Capacity
I. Foundations
J. Consolidation & Settlement
K. Seepage & Flow Nets
L. Slopes
87.$A$saturated$soil$sample$from$field$is$tested$and$the$specific$gravity$is$obtained$as$2.85$with$total$unit$mass$2535$kg/m3.$Calculate$
the$dry$unit$mass.$
(A)$2265$kg$
(B)$2365$kg$
(C)$2465$kg$
(D)$2565$kg$
$
$
$
$
$
Question$88>90$are$based$on$the$following$figure.$The$section$of$a$permanent$retaining$wall$is$shown$below.$Six$meters$excavation$
was$done$through$two$different$soil$layers.$

γ1 = 19kN/m3
Layer 1 φ1' = 33¢X 3m
c1' = 0

γ2 = 20kN/m3
φ2' = 25¢X
Layer 2 3m
c2' = 0

$
$

123
88.$Calculate$the$active$earth$pressure$coefficient$ K a $for$each$layer$based$on$each$friction$angle$ φ .$

(A)$0.4553$for$layer$1$and$0.5774$for$layer$2$
(B)$0.2948$for$layer$1$and$0.4059$for$layer$2$
(C)$3.3921$for$layer$1$and$2.4639$for$layer$2$
(D)$0.5430$for$layer$1$and$0.6371$for$layer$2$
$
$
89.$Calculate$the$total$lateral$pressure$against$the$retaining$wall$at$depth$of$6$m.$
(A)$35$kN/m2$
(B)$45$kN/m2$
(C)$55$kN/m2$
(D)$65$kN/m2$
$
$
90.$A$sample$of$soil$has$a$mass$of$17.4$gram,$volume$of$9$cm3,$oven>dry$mass$of$14.6$gram,$and$specific$gravity$of$2.1.$Determine$the$
void$ratio$of$the$soil$based$on$the$following$information.$
(A)$0.3$
(B)$0.4$
(C)$0.5$
(D)$0.6$
$
$ $

124
$ $

125
87.$The$correct$answer$is$(B).$
Assume$that$the$total$volume$of$saturated$soil$(soil$+$water)$is$1$m3$→$ Vs + Vw = 1$

Substitute$the$volume$relationship$to$this$equation:$
$
ms + mw = 2535
GsVs ρ w + Vw ρ w = 2535
2.85 × Vs ×1000 + (1 − Vs ) ×1000 = 2535
$$
2850Vs + 1000 − 1000Vs = 2535
1850Vs = 1535
Vs = 0.83m3
$
After$obtaining$volume$of$soil,$we$can$calculate$dry$unit$mass$(mass$of$soil).$
$
ms = GsVs ρ w
ms = 2.85 × 0.83 ×1000 $$
ms = 2365.5kg
$
$
88.$The$correct$answer$is$(B).$
Based$on$Rankine$theory,$
" φ# " 33° #
K a1 = tan 2 % 45° − & = tan 2 % 45° − & = 0.2948
' 2( ' 2 (
$$
" φ # " 25° #
K a 2 = tan 2 % 45° − & = tan 2 % 45° − & = 0.4059
' 2( ' 2 (

126
89.$The$correct$answer$is$(D).$
Total$lateral$pressure$is$the$combination$between$active$earth$pressure$and$pore$water$/$hydrostatic$pressure.$
$
Active$earth$pressure$at$depth$=$6$m:$
$
σ a = K a (γ 1 z1 + (γ 2 − γ w ) z2 )
σ a = 0.4059 (19 × 3 + ( 20 − 9.8 ) × 3) $$
σ a = 35.56kN / m2
$
Pore$water$/$hydrostatic$pressure$at$depth$=$6$m:$
$
σ w = γ w z2
σ w = 9.8 × 3 $
σ w = 29.4kN / m2
$
Total$lateral$pressure$at$depth$=$6$m:$
σ total = σ a + σ w
σ total = 35.56 + 29.4 $$
σ total = 64.96kN / m2 ≈ 65kN / m2
$
$
$
$

127
90.$The$correct$answer$is$(A).$
The$sample$of$soil$can$be$divided$into$three$components:$dry$soil,$water,$and$air.$
Dry$soil$properties:$
$
ms 14.6
Vs = = = 6.95cm3 $$
Gs ρ w 2.1×1
$
$
Volume$of$the$void$can$be$defined$as$the$summation$between$volume$of$water$and$air.$Therefore,$
$
Vv = Vtotal − Vs = 9 − 6.95 = 2.05cm3 $$

$
$
Void$ratio$is$calculated$by$using$this$following$equation:$
$
Vv 2.05
e= = = 0.29 ≈ 0.3 $$
Vs 6.95
$ $

128
Chapter 15

Transportation
Engineering

A. Geometric Design (Streets, Highways)


B. Geometric Design (Intersections)
C. Pavement
D. Traffic (Safety)
E. Traffic (Capacity)
F. Traffic Flow
G. Traffic (Control Devices)
H. Transportation Modeling
Question$91$and$92$are$based$on$the$following$information.$
The$relationship$of$traffic$flow$can$be$written$as$ q = kv $$
Where$ q $is$the$traffic$volume,$ k $is$the$traffic$density,$ v $is$the$average$speed$of$vehicles.$
Based$on$several$tests$and$observations,$ v $can$be$approximated$by$using$this$following$equation.$
v = 80 − 0.5k $$
$
$
$
91.$Determine$the$maximum$capacity$of$traffic$density$at$rush$hour$if$the$average$speed$of$vehicles$is$30$km/hour.$
(A)$70$vehicles/km$
(B)$80$vehicles/km$
(C)$90$vehicles/km$
(D)$100$vehicles/km$
$
$
$
92.$Find$the$maximum$capacity$of$total$traffic$volume$of$the$road.$
(A)$3000$vehicles/hour$
(B)$3200$vehicles/hour$
(C)$3400$vehicles/hour$
(D)$3600$vehicles/hour$
$
$

130
Question$93$and$94$are$based$on$this$following$figure.$
A$vertical$curve$and$its$complete$data$are$shown$in$the$figure$below.$
L = 12 sta

PVC EVC
-2% +1.8%

PVI = sta 76+00


PVI elevation = 500 m $
$
$
93.$Find$the$low$point$station$for$this$vertical$curve.$
(A)$sta$75+60$
(B)$sta$76+00$
(C)$sta$76+32$
(D)$sta$76+40$
$
$
94.$Determine$the$elevation$of$the$low$point.$
(A)$502.68$m$
(B)$503.68$m$
(C)$504.68$m$
(D)$505.68$m$
$
$ $

131
$ $

132
91.$The$correct$answer$is$(D).$
By$using$the$equation$of$average$speed$of$vehicles,$we$can$obtain$the$traffic$density$at$the$rush$hour.$
$
v = 80 − 0.5k
30 = 80 − 0.5k $
k = 100 vehicles/km
$
$
$
92.$The$correct$answer$is$(B).$
By$combining$two$equations$given$in$the$above$information,$we$can$obtain$a$new$equation$about$the$relationship$between$the$
traffic$volume$and$traffic$density.$
$
q = kv
q = k (80 − 0.5k ) $
q = 80k − 0.5k 2
$
According$to$mathematics$principle,$we$can$get$the$maximum$value$of$ q $when$the$first$derivation$of$the$equation$is$equal$to$0.$
$
q = 80k − 0.5k 2
dq
= 80 − k = 0 → k = 80 vehicles/km $
dk
2
qmax = 80 (80 ) − 0.5 (80 ) = 3200 vehicles/hour
$

133
93.$The$correct$answer$is$(C).$
L
PVC = PVI −
2
12 sta
PVC = sta 76+00 − $$
2
PVC = sta 70+00
$
G1 L
xm =
G1 − G2

xm =
( −0.02 )(12 sta ) $$
−0.02 − 0.018
xm = 6.32 sta
$
lowpoint = PVC + xm
lowpoint = sta 70+00 + 6.32 sta $$
lowpoint = sta 76+32
$
94.$The$correct$answer$is$(D).$
$

( G2 − G1 ) xm 2
ellow point = elPVC + G1 xm +
2L
2

ellow point = ( 500 + 0.02 × 6 sta ) + ( −0.02 )( 6.32 sta ) +


( 0.018 + 0.02 )( 6.32sta ) $
2 (12 sta )
ellow point = 505.68m

$ $

134
Chapter 16

Environmental
Engineering

A. Water Quality
B. Basic Water & Air Tests
C. Regulations
D. Supply & Treatment of Water
E. Wastewater
Question$95$and$96$are$based$on$the$following$information.$
A$sample$of$wastewater$is$placed$in$the$incubator$by$keeping$the$temperature$at$23°C.$After$5$days,$$the$BOD$(Biochemical$Oxygen$
Demand)$is$found$to$be$234$mg/L.$Assume$that$the$reaction$$rate$constant$is$0.13$d>1$(base$e).$
$
$
95.$Determine$the$ultimate$BOD$of$the$sample.$
(A)$470$mg/L$
(B)$480$mg/L$
(C)$490$mg/L$
(D)$500$mg/L$
$
$
$
96.$Determine$the$BOD$of$the$sample$if$the$incubation$period$is$7$days.$
(A)$280$mg/L$
(B)$290$mg/L$
(C)$300$mg/L$
(D)$310$mg/L$
$
$
$
$

136
97.$A$sample$of$freshwater$is$taken$from$a$small$river.$After$several$observations,$the$sample$is$found$to$contain$dissolved$oxygen$
concentration$of$6.2$mg/L$when$the$temperature$is$24.6°C$and$the$atmospheric$pressure$is$740$mmHg.$A$partial$listing$of$the$
solubility$of$dissolved$oxygen$in$freshwater$at$equilibrium$with$dry$air$containing$21.7%$oxygen$and$at$an$atmospheric$
pressure$of$760$mmHg$is$as$follows.$
$
temperature$ oxygen$solubility$
(°C)$ (mg/L)$
22$ 9.6$
23$ 9.3$
24$ 8.9$
25$ 8.6$
26$ 8.2$
!
!
Find$the$saturation$of$dissolved$oxygen$in$the$sample$water.$
(A)$50%$
(B)$60%$
(C)$70%$
(D)$80%$
$
$
$

137
98.$A$tank$reactor$treats$0.35$m3/s$of$settled$wastewater$having$258$mg/L$BOD5$at$22°C.$The$design$mean$cell$resistance$time,$ θ cd ,$is$

12$days.$
Vss ,mg
Y= = 0.55 $$
BOD5,mg

$
The$ effluent$ BOD5$ is$ 6.4$ mg/L.$ MLVSS$ =$ 3600$ mg/L.$ The$ endogenous$ decay$ coefficient$ K d = 0.06d −1 .$ Determine$ the$ reactor$

capacity.$
(A)$6350$m3/day$
(B)$6420$m3/day$
(C)$7210$m3/day$
(D)$8110$m3/day$
$
$
$ $

138
$ $

139
95.$The$correct$answer$is$(C).$
Substitute$the$five$days$values$into$the$equation$for$BOD.$
$

(
yt = L 1 − e− kt )
(
234 = L 1 − e−(0.13)(5) ) $$
L = 489.6 mg/L ≈ 490 mg/L
$
$
$
$
$
96.$The$correct$answer$is$(B).$
Substitute$the$seven$days$values$into$the$equation$for$BOD.$
$

(
yt = L 1 − e − kt )
(
yt = 489.6 1 − e −( 0.13)( 7 ) ) $$
yt = 292.5 mg/L ≈ 290 mg/L
$
$
$
$
$

140
97.$The$correct$answer$is$(C).$
The$dissolved$oxygen$(DO)$at$760$mmHg$can$be$found$by$linear$interpolation$from$the$table.$
$
DO24.6°C − 8.9 24.6 − 24
=
8.6 − 8.9 25 − 24
DO24.6°C − 8.9 = 0.6 × ( −0.3) $$
DO24.6°C = 8.72 mg/L
$
Oxygen$ is$ only$ slightly$ soluble$ in$ water$ and$ does$ not$ react$ with$ water.$ Therefore,$ Henry’s$ law$ is$ applicable,$ and$ oxygen’s$
solubility$is$directly$proportional$to$its$partial$pressure.$
$
! 740 "! 6.2 "
%saturation= % &% & ×100% = 69.2% ≈ 70% $$
' 760 (' 8.72 (
$
$
$
$
$
$
$
$
$
$
$

141
98.$The$correct$answer$is$(D).$
The$solids$residence$time$is$
θ =V / Q$
$
The$suspended$solids$concentration$is$determined$using$the$following$equation.$
$
θcd Y ( So − S )
X=
θ (1 + K dθcd )
$
$
Combining$equations,$the$volume$of$the$reactor$can$be$found.$
$
θcd QY ( So − S )
V=
(
X 1 + K dθcd )
12 × 0.35 × 0.55 × ( 258 − 6.4 )
V= $ $
3600 × (1 + 0.06 ×12 )
V = 0.094m3 / s
V = 8110m3 / day

142
Chapter 17

Construction

A. Documentation
B. Procurement
C. Project Delivery
D. Operations & Methodology
E. Scheduling
F. Project Management
G. Safety
H. Estimation
99.$What$is$the$definition$of$critical$path$in$a$project?$
(A)$The$longest$path$of$sequential$tasks$through$a$project$
(B)$The$time$unaccounted$for$in$a$project$
(C)$The$extra$time$available$in$a$project$
(D)$The$sequences$of$tasks$in$a$project$
$
$
$
$
100.$$The$illustrated$model$below$is$one$of$project$scheduling$tools,$which$is$called$as$….$
$

$
(A)$a$PERT$chart$
(B)$a$bar$(Gantt)$chart$
(C)$Activity>On>Node$(AON)$diagram$
(D)$Activity>On>Arrow$(AOA)$diagram$
$
$

144
Question$101>104$can$be$solved$by$using$this$following$diagram.$

C
2

A F
3 3
D
START 3 FINISH

B E
5 4
$
Note$that$the$duration$in$days$is$displayed$below$the$task.$
$
$
$
101.$Determine$the$critical$path$of$the$diagram.$$
(A)$START>A>C>F>FINISH$
(B)$START>A>D>F>FINISH$
(C)$START>B>D>F>FINISH$
(D)$START>B>E>FINISH$
$
$
$
$
$
$

145
102.$Find$the$float$time$of$activity$D.$
(A)$0$day$
(B)$1$day$
(C)$2$days$
(D)$3$days$
$
$
$
103.$Find$the$EST$(Earliest$Start$Time)$of$activity$F.$
(A)$day$8$
(B)$day$9$
(C)$day$10$
(D)$day$11$
$
$
$
$
104.$Determine$the$LST$(Latest$Start$Time)$of$activity$C.$
(A)$day$5$
(B)$day$6$
(C)$day$7$
(D)$day$8$ $

146
$
$ $

147
99.$The$correct$answer$is$(A).$
$
$
100.$The$correct$answer$is$(C).$
$
$
101.$The$correct$answer$is$(C).$
$
Critical$path$is$defined$as$the$longest$duration$in$a$project.$
From$the$AON$diagram,$we$could$determine$the$total$duration$of$sequences$for$each$path.$
START>A>C>F>FINISH$ :$3$+$2$+$3$=$8$days$
START>A>D>F>FINISH$ :$3$+$3$+$3$=$9$days$
START>B>D>F>FINISH$ :$5$+$3$+$3$=$11$days$
START>B>E>FINISH$ $ :$5$+$4$=$9$days$
Therefore,$we$can$conclude$that$path$START>B>D>F>FINISH$is$the$critical$path$of$this$project.$
$
$
$
$
$
$
$
$

148
102.$The$correct$answer$is$(A).$
$
Activity$D$is$along$the$critical$path$(please$see$the$solution$for$number$103),$so$that$the$float$time$(TF)$of$activity$D$is$0$day.$
The$project$scheduling$can$be$summarized$in$this$diagram.$
$
Activity$ Duration$ EST$ EFT$ LST$ LFT$ TF$
(days)$ (day$no.)$ (day$no.)$ (day$no.)$ (day$no.)$ (days)$
A$ 3$ 0$ 3$ 3$ 6$ 3$
B$ 5$ 0$ 5$ 0$ 5$ 0$
C$ 2$ 3$ 5$ 6$ 8$ 3$
D$ 3$ 5$ 8$ 5$ 8$ 0$
E$ 4$ 5$ 9$ 7$ 11$ 2$
F$ 3$ 8$ 11$ 8$ 11$ 0$
$
$
$
$
103.$The$correct$answer$is$(A).$
Activity$F$is$affected$by$two$activities:$activity$C$and$D.$Therefore,$the$EST$(Earliest$Start$Time)$of$activity$F$can$be$determined$
as$the$maximum$EFT$of$the$two$activities.$
$
$
$

149
104.$The$correct$answer$is$(B).$
Activity$C$is$only$affected$by$activity$A.$Therefore,$the$LST$(Latest$Start$Time)$of$activity$C$is$the$same$as$the$LFT$of$the$activity$
A.$ $

150
Chapter 18

Surveying

A. Geometry & Trigonometry


B. Area Calculations
C. Earth & Volume Calculations
D. Closure
E. Coordinates
F. Leveling
105.$Find$the$length$of$the$curve$AB$based$on$the$following$illustration.$
$
$

θ = 12¢X

A B

R = 1150 ft R = 1150 ft

$ O $
(A)$220$ft$
(B)$240$ft$
(C)$260$ft$
(D)$280$ft$
$
$
$
$
$
$
$
152
106.$Determine$the$approximate$hatched$area$in$the$following$figure.$
$
$

20.1 m
18.5 m
17.6 m
14.3 m 15.2 m

6m 6m 6m 6m
$
(A)$410$m2$
(B)$430$m2$
(C)$450$m2$
(D)$470$m2$
$
$
$
$
$
$
$

153
Question$107$and$108$are$based$on$the$following$figure.$
$
O

A B
L = 250 m

α = 106¢X
30'

$
$
107.$Determine$the$approximate$tangent$distance.$
(A)$210$m$
(B)$215$m$
(C)$220$m$
(D)$225$m$
$
$
108.$Determine$the$approximate$length$of$curve$connecting$point$A$and$B.$
(A)$260$m$
(B)$270$m$
(C)$280$m$
(D)$290$m$
$

154
The$following$data$is$used$to$solve$question$109$and$110.$
An$earthwork$of$new$roadway$requires$less$soil$to$be$thrown$out,$so$that$engineers$should$calculate$the$cut>and>fill$volume$based$
on$this$data.$Note:$it$can$be$assumed$that$for$the$transition$region$from$fill$area$to$cut$area,$both$cut$and$fill$area$are$triangular.$
$
Station$ Cut$area$ Fill$area$
(m)$ (m2)$ (m2)$
10+00$ >$ 153.42$
10+4.50$ >$ 32.56$
10+12.35$ 13.67$ 8.25$
10+16.40$ 52.84$ >$
10+20$ 165.14$ >$
$$ $
109.$Determine$the$total$volume$of$fill$work.$
(A)$500$m3$
(B)$505$m3$
(C)$510$m3$
(D)$515$m3$
$
110.$Determine$the$total$volume$of$cut$work.$
(A)$540$m3$
(B)$560$m3$
(C)$580$m3$
(D)$600$m3$

155
$
$
$ $

156
105.$The$correct$answer$is$(B).$
$
Lcurve = R × α
$ π %
Lcurve = 1150 × 12° & ' $$
( 180° )
Lcurve = 240.8 ft ; 240 ft
$ $
$
$
$
106.$The$correct$answer$is$(B).$
We$can$adopt$the$trapezoidal$rule$to$get$the$total$area$of$hatched$area.$
$
!h +h "
A = w # 1 5 + h2 + h3 + h4 $
% 2 &
! 14.3 + 15.2 "
A = 6# + 20.1 + 18.5 + 17.6 $ $$
% 2 &
A = 425.7m 2 ≈ 430m 2
$
$
$
$
$
$

157
107.$The$correct$answer$is$(A).$
$
" 1° #
α = 106° + 30 ' $ % = 106.5°
& 60 ' ' $
$
L
T=
"α #
2 cos $ %
&2'
250
T= $
" 106.5° #
2 cos $ %
& 2 '
T = 208.9m ≈ 210m
$
$
$
$
$
$
$
$
$
$
$
$
$

158
108.$The$correct$answer$is$(D).$
The$length$of$curve$can$be$solved$by$multiplying$radius$of$the$curve$with$the$intersection$angle.$
# π $
α = 106.5° % & = 1.8588rad
' 180° ( $
$
L
R=
"α #
2sin $ %
&2'
250
R= $$
" 106.5° #
2sin $ %
& 2 '
R = 156m
$
Lcurve = R × α
Lcurve = 156 ×1.8588 $$
Lcurve = 289.97m ; 290m
$
$
$
$
$
$
$
$
$

159
109.$The$correct$answer$is$(D).$
As$it$is$written$in$the$question$that$the$cut$and$fill$area$can$be$assumed$as$triangular$in$shape$for$the$transition$area.$Then$for$
the$other$area,$the$cut$and$fill$area$are$assumed$as$trapezoidal$area.$Total$volume$for$cut$and$fill$work$are$summarized$in$the$
following$table.$
Cut$ Fill$ Cut$volume$ Fill$volume$
Station$
area$ area$ (m3)$ (m3)$
(m)$
(m2)$ (m2)$
10+00$ >$ 153.42$ $ $
! 153.42 + 20.56 "
$ $ $ >$ 4.50 # $ = 391.46 $
% 2 &
10+4.50$ >$ 20.56$ $ $
! 13.67 " ! 20.56 + 8.25 "
$ $ $ 7.85 # $ = 35.77 $ 7.85 # $ = 113.08 $
% 3 & % 2 &
10+12.35$ 13.67$ 8.25$ $ $
! 13.67 + 52.84 " ! 8.25 "
$ $ $ 4.05 # $ = 134.68 $ 4.05 # $ = 11.14 $
% 2 & % 3 &
10+16.40$ 52.84$ >$ $ $
! 52.84 + 165.14 "
$ $ $ 3.6 # $ = 392.36 $ >$
% 2 &
10+20$ 165.14$ >$ $ $
$ $ total$ 562.81! 515.68!
$
The$total$volume$of$fill$work$is$515.68$m3$(515$m3).$

160
$
110.$The$correct$answer$is$(B).$
Referring$to$the$table$above,$the$total$volume$of$cut$work$is$562.81$m3$(560$m3).$
$

161

You might also like